GMAT Exam  >  GMAT Tests  >  Practice Questions for GMAT  >  Test: Cause and Effect - GMAT MCQ

Test: Cause and Effect - GMAT MCQ


Test Description

10 Questions MCQ Test Practice Questions for GMAT - Test: Cause and Effect

Test: Cause and Effect for GMAT 2024 is part of Practice Questions for GMAT preparation. The Test: Cause and Effect questions and answers have been prepared according to the GMAT exam syllabus.The Test: Cause and Effect MCQs are made for GMAT 2024 Exam. Find important definitions, questions, notes, meanings, examples, exercises, MCQs and online tests for Test: Cause and Effect below.
Solutions of Test: Cause and Effect questions in English are available as part of our Practice Questions for GMAT for GMAT & Test: Cause and Effect solutions in Hindi for Practice Questions for GMAT course. Download more important topics, notes, lectures and mock test series for GMAT Exam by signing up for free. Attempt Test: Cause and Effect | 10 questions in 20 minutes | Mock test for GMAT preparation | Free important questions MCQ to study Practice Questions for GMAT for GMAT Exam | Download free PDF with solutions
Test: Cause and Effect - Question 1

Physician: In itself, exercise does not cause heart attacks; rather, a sudden increase in an exercise regimen can be a cause. When people of any physical condition suddenly increase their amount of exercise, they also increase their risk of heart attack. As a result, there will be an increased risk of heart attack among employees of this company due to the new health program.

The conclusion drawn by the physician follows logically if which one of the flowing is assumed?

Detailed Solution for Test: Cause and Effect - Question 1

(A) Employees will abruptly increase their amount of exercise as a result of the new health program.
This assumption aligns with the physician's argument that a sudden increase in exercise regimen can increase the risk of heart attacks. If employees do indeed abruptly increase their exercise due to the new health program, then the increased risk of heart attacks among employees would be a logical consequence. Thus, this assumption supports the conclusion.

(B) The exercises involved in the new health program are more strenuous than those in the previous health program.
This assumption is not necessary for the conclusion drawn by the physician. The physician's argument focuses on the sudden increase in exercise amount, rather than the relative intensity of the exercises. Therefore, this assumption is not required for the conclusion to follow.

(C) The new health program will force employees of all levels of health to exercise regularly.
This assumption is also not necessary for the conclusion to follow. The physician's argument does not depend on whether all employees, regardless of their health levels, participate in the new health program. It focuses on the sudden increase in exercise amount and the associated risk of heart attacks. Therefore, this assumption is not required.

(D) The new health program constitutes a sudden change in the company's policy.
This assumption aligns with the physician's argument that sudden changes in exercise regimen can increase the risk of heart attacks. If the new health program indeed represents a sudden change in the company's policy, then the argument that there will be an increased risk of heart attacks among employees becomes more plausible. Therefore, this assumption supports the conclusion.

(E) All employees, no matter what their physical condition, will participate in the new health program.
This assumption is not necessary for the conclusion to follow. The physician's argument does not require the participation of all employees, regardless of their physical condition, in the new health program. The focus is on the increased risk associated with a sudden increase in exercise amount. Hence, this assumption is not required.

Therefore, the assumption that is necessary for the physician's conclusion to follow logically is (A) Employees will abruptly increase their amount of exercise as a result of the new health program.

Test: Cause and Effect - Question 2

Most antidepressant drugs cause weight gain. While dieting can help reduce the amount of weight gained while taking such antidepressants, some weight gain is unlikely to be preventable.

The information above most strongly supports which one of the following?

Detailed Solution for Test: Cause and Effect - Question 2

A. A physician should not prescribe any antidepressant drug for a patient if that patient is overweight.
This option is not directly supported by the information provided. The information states that most antidepressant drugs cause weight gain and that some weight gain may be unavoidable even with dieting. It does not suggest that overweight patients should not be prescribed antidepressants altogether.

B. People who are trying to lose weight should not ask their doctors for an antidepressant drug.
The information does not directly support this option. While it mentions that antidepressant drugs can cause weight gain, it does not imply that people trying to lose weight should completely avoid asking their doctors for these medications.

C. At least some patients taking antidepressant drugs gain weight as a result of taking them.
This option is strongly supported by the given information. The statement explicitly mentions that most antidepressant drugs cause weight gain, indicating that weight gain is a likely outcome for at least some patients taking these medications.

D. The weight gain experienced by patients taking antidepressant drugs should be attributed to lack of dieting.
This option is not supported by the information provided. The statement acknowledges that dieting can help reduce the amount of weight gained while taking antidepressant drugs but implies that some weight gain is still likely to occur. It does not attribute weight gain solely to a lack of dieting.

E. All patients taking antidepressant drugs should diet to maintain their weight.
This option is not supported by the information. While the statement mentions that dieting can help reduce the amount of weight gained while taking antidepressant drugs, it does not suggest that all patients should diet to maintain their weight. The information does not address the specific dietary needs or circumstances of individual patients.

Based on the given information, the option that is most strongly supported is C. At least some patients taking antidepressant drugs gain weight as a result of taking them.

1 Crore+ students have signed up on EduRev. Have you? Download the App
Test: Cause and Effect - Question 3

A small company with a radical four-day workweek system is considering changing its policy. Currently, this company requires all employees to arrive at work in the company's office Monday through Thursday; they need not work Friday. The proposed policy would permit each employee every week to choose one day from Monday to Friday not to work.

The adoption of this policy would be most likely to decrease employees' productivity if the employees' job functions required them to

Detailed Solution for Test: Cause and Effect - Question 3

A. work without interruption from other employees
If employees work without interruption from other employees, the ability to choose one day not to work would not necessarily affect their productivity. They can still maintain focus and complete their tasks efficiently. Therefore, this job function would not be significantly impacted by the policy change.

B. consult at least once a day with employees from other companies
If employees need to consult with employees from other companies on a daily basis, the ability to choose one day not to work could disrupt the regular flow of communication and collaboration. This could lead to delays or missed opportunities for important consultations, potentially decreasing productivity. Therefore, this job function would likely be impacted by the policy change.

C. submit their work for a supervisor's eventual approval
If employees need to submit their work for a supervisor's approval, the ability to choose one day not to work could cause delays in the approval process. This could potentially slow down overall progress and productivity. Therefore, this job function would likely be impacted by the policy change.

D. interact frequently with each other throughout the entire workday
If employees need to interact frequently with each other throughout the entire workday, the ability to choose one day not to work could disrupt the team dynamics and collaboration. The absence of certain team members on different days could hinder effective communication and coordination, potentially decreasing productivity. Therefore, this job function would likely be impacted by the policy change.

E. undertake projects that take several days to complete
If employees undertake projects that take several days to complete, the ability to choose one day not to work would not necessarily hinder their productivity. As long as the projects can be effectively managed and progress tracked, the policy change may not significantly impact their ability to complete their work. Therefore, this job function would not be greatly impacted by the policy change.

Based on the analysis, the job function most likely to see a decrease in productivity with the proposed policy change is:

D. interact frequently with each other throughout the entire workday

This job function heavily relies on frequent interaction and collaboration among team members. The policy change, which allows employees to choose different days not to work, could disrupt the team dynamics and hinder effective communication, resulting in decreased productivity.

Test: Cause and Effect - Question 4

Researchers took a group of teenagers who had never smoked and for one year tracked whether they took up smoking and how their mental health changed. Those who began smoking within a month of the study's start were four times as likely to be depressed at the study's end than those who did not begin smoking. Since nicotine in cigarettes changes brain chemistry, perhaps thereby affecting mood, it is likely that smoking contributes to depression in teenagers.

Which of the following, if true, most strengthens the argument?

Detailed Solution for Test: Cause and Effect - Question 4

(A) Participants who were depressed at the study's start were no more likely to be smokers at the study's end than those who were not depressed.
This option does not provide additional support for the argument. It suggests that depression at the study's start did not affect smoking behavior, but it does not directly strengthen the link between smoking and depression.

(B) Participants who began smoking within a month of the study's start were no more likely than those who began midway through to have quit smoking by the study's end.
This option is not directly relevant to the argument. It focuses on the quitting behavior of those who began smoking at different times, rather than strengthening the link between smoking and depression.

(C) Few, if any, of the participants in the study were friends or relatives of other participants.
This option is not relevant to the argument. The relationship between participants in the study does not have a direct impact on the link between smoking and depression.

(D) Some participants entered and emerged from a period of depression within the year of the study.
This option weakens the argument rather than strengthening it. If some participants entered and emerged from depression within the study period, it suggests that depression may not be solely caused by smoking but could have other contributing factors. Therefore, this option does not support the argument.

(E) The researchers did not track use of alcohol by the teenagers.
This option does not directly strengthen the argument regarding the link between smoking and depression. It introduces a new factor (alcohol use) that is not directly related to the impact of smoking on depression.

Based on the analysis, none of the options provided directly strengthen the argument. However, the most relevant option is:

(A) Participants who were depressed at the study's start were no more likely to be smokers at the study's end than those who were not depressed.

Test: Cause and Effect - Question 5

Y has been believed to cause Z. A new report, noting that Y and Z are often observed to be preceded by X, suggests that X, not Y, may be the cause of Z.

Which of the following further observations would best support the new report’s suggestion?

Detailed Solution for Test: Cause and Effect - Question 5

(A) In cases where X occurs but Y does not, X is usually followed by Z.
This option supports the new report's suggestion. If X occurring without Y still results in Z, it suggests that X is a sufficient cause of Z, independent of Y.

(B) In cases where X occurs, followed by Y, Y is usually followed by Z.
This option is not aligned with the new report's suggestion. It suggests that Y is followed by Z, which doesn't support the claim that X, not Y, may be the cause of Z.

(C) In cases where Y occurs but X does not, Y is usually followed by Z.
This option is not aligned with the new report's suggestion. It suggests that Y is followed by Z, which doesn't provide support for the claim that X, not Y, may be the cause of Z.

(D) In cases where Y occurs but Z does not, Y is usually preceded by X.
This option does not support the new report's suggestion. It suggests that Y is usually preceded by X, but it doesn't provide evidence about the relationship between X and Z.

(E) In cases where Z occurs, it is usually preceded by X and Y.
This option does not directly support the new report's suggestion. It provides information about the sequence of X, Y, and Z when Z occurs but does not address the relationship between X and Z independent of Y.

Based on the analysis, the option that best supports the new report's suggestion is:

(A) In cases where X occurs but Y does not, X is usually followed by Z.

This option supports the claim that X may be the cause of Z, as X occurring without Y still results in Z.

Test: Cause and Effect - Question 6

Many institutions of higher education suffer declining enrollments during periods of economic slowdown. At two-year community colleges, however, enrollment figures boom during these periods when many people have less money and there is more competition for jobs.

Each of the following, if true, helps to explain the enrollment increases in two-year community colleges described above EXCEPT:

Detailed Solution for Test: Cause and Effect - Question 6

A. During periods of economic slowdown, two-year community colleges are more likely than four-year colleges to prepare their students for the jobs that are still available.
This option helps explain the enrollment increases in two-year community colleges. It suggests that students perceive two-year community colleges as better equipped to provide them with job preparation during economic slowdowns.

B. During periods of economic prosperity, graduates of two-year community colleges often continue their studies at four-year colleges.
This option does not directly explain the enrollment increases in two-year community colleges during economic slowdowns. It addresses the actions of graduates during periods of economic prosperity but does not provide insight into the enrollment increases during economic slowdowns.

C. Tuition at most two-year community colleges is a fraction of that at four-year colleges.
This option helps explain the enrollment increases in two-year community colleges. It suggests that lower tuition at two-year community colleges makes them a more affordable option for individuals during economic slowdowns.

D. Two-year community colleges devote more resources than do other colleges to attracting those students especially affected by economic slowdowns.
This option helps explain the enrollment increases in two-year community colleges. It indicates that two-year community colleges focus on attracting students who are particularly affected by economic slowdowns, which can contribute to higher enrollments.

E. Students at two-year community colleges, but not those at most four-year colleges, can control the cost of their studies by choosing the number of courses they take each term.
This option helps explain the enrollment increases in two-year community colleges. It highlights the flexibility of course selection and cost control available to students at two-year community colleges, making them more appealing during economic slowdowns.

Test: Cause and Effect - Question 7

One summer, floods covered low-lying garlic fields situated in a region with a large mosquito population. Since mosquitoes lay their eggs in standing water, flooded fields would normally attract mosquitoes, yet no mosquitoes were found in the fields. Diallyl sulfide, a major component of garlic, is known to repel several species of insects, including mosquitoes, so it is likely that diallyl sulfide from the garlic repelled the mosquitoes.

Which of the following, if true, most strengthens the argument?

Detailed Solution for Test: Cause and Effect - Question 7

(A) Diallyl sulfide is also found in onions but at concentrations lower than in garlic.
This option weakens the argument. If onions contain diallyl sulfide, albeit at lower concentrations, it suggests that the absence of mosquitoes in the flooded garlic fields may not be solely attributed to diallyl sulfide.

(B) The mosquito population of the region as a whole was significantly smaller during the year in which the flooding took place than it had been in previous years.
This option weakens the argument. If the mosquito population as a whole was smaller during the year of flooding, it provides an alternative explanation for the absence of mosquitoes in the flooded garlic fields, which may not necessarily be due to the repelling effect of diallyl sulfide.

(C) By the end of the summer, most of the garlic plants in the flooded fields had been killed by waterborne fungi.
This option weakens the argument. If the garlic plants were killed by waterborne fungi, it raises the possibility that the absence of mosquitoes in the flooded garlic fields was due to the unfavorable conditions caused by the fungi rather than the repelling effect of diallyl sulfide.

(D) Many insect species not repelled by diallyl sulfide were found in the flooded garlic fields throughout the summer.
This option strengthens the argument. If other insect species, which are not repelled by diallyl sulfide, were found in the flooded garlic fields, it suggests that the absence of mosquitoes in the fields may indeed be attributed to the repelling effect of diallyl sulfide.

(E) Mosquitoes are known to be susceptible to toxins in plants other than garlic, such as marigolds.
This option is not directly relevant to the argument. It discusses the susceptibility of mosquitoes to toxins in other plants but does not provide additional support for the claim that diallyl sulfide from garlic repelled the mosquitoes.

Based on the analysis, the option that most strengthens the argument is:

(D) Many insect species not repelled by diallyl sulfide were found in the flooded garlic fields throughout the summer.

This option supports the argument by highlighting that other insect species, which are not repelled by diallyl sulfide, were present in the flooded garlic fields. This reinforces the idea that the absence of mosquitoes in the fields was specifically due to the repelling effect of diallyl sulfide.

Test: Cause and Effect - Question 8

When a female fruit fly is placed in a cage with several potential mates, her offspring tend to be stronger than when she is caged with only a single male of average strength. Therefore, the female fruit fly must be able to determine which mate will give her the strongest offspring.

Which of the following, if true, is the best basis for a counterargument against the conclusion drawn above?

Detailed Solution for Test: Cause and Effect - Question 8

(A) The offspring also tend to be stronger when, in a cage containing several female and male fruit flies, the males vary widely in strength and size, from very small and weak to very large and strong.

This option suggests that when there is a wide variation in the strength and size of males in the cage, the offspring tend to be stronger. This implies that the female fruit fly does not have the ability to accurately choose the mate that will produce the strongest offspring.

(B) Given a choice between a male fruit fly of average strength and a relatively weak male fruit fly, the female will mate with the male of average strength.

This option implies that the female fruit fly will prefer to mate with a male of average strength rather than a weak male. This challenges the idea that the female fruit fly can consistently choose the mate that will produce the strongest offspring.

(C) When caged with just one male fruit fly of average strength, the female produces no fewer offspring than when caged with a variety of potential mates.

This option states that when the female fruit fly is caged with only one male of average strength, the number of offspring produced is not lower compared to when she is caged with a variety of potential mates. This suggests that the presence of multiple potential mates does not necessarily result in stronger offspring.

(D) Strong male fruit flies produce stronger offspring than weak male fruit flies, provided that both are mated to females of equal strength.

This option highlights that strong male fruit flies produce stronger offspring when mated with females of equal strength. While this information is relevant, it does not directly address the ability of the female fruit fly to determine which mate will give her the strongest offspring.

(E) In a cage containing several male fruit flies, the strongest male does not allow the female to mate with any of the other males.

This option states that the strongest male in a cage does not allow the female to mate with any other males present. This suggests that the female fruit fly's ability to choose the strongest mate is restricted by the actions of the strongest male, thus challenging the idea that the female can accurately select the mate that will produce the strongest offspring.

Based on these explanations, option (E) provides the best basis for a counterargument against the conclusion drawn in the initial statement.

Test: Cause and Effect - Question 9

In the past most airline companies minimized aircraft weight to minimize fuel costs. The safest airline seats were heavy, and airlines equipped their planes with few of these seats. This year the seat that has sold best to airlines has been the safest one - a clear indication that airlines are assigning a higher priority to safe seating than to minimizing fuel costs.

Which of the following, if true, most seriously weakens the argument above?

Detailed Solution for Test: Cause and Effect - Question 9

The option that most seriously weakens the argument above is (E) Because of technological innovations, the safest airline seat on the market this year weighed less than most other airline seats on the market.

The argument states that the best-selling airline seat this year is the safest one, implying that airlines are prioritizing safety over minimizing fuel costs. However, option (E) weakens this argument by introducing the fact that the safest seat on the market is lighter than most other seats due to technological innovations.

If the safest seat is lighter, it means that airlines can prioritize safety without significantly increasing the weight of the aircraft. This suggests that the decision to choose the safest seat may not necessarily be a trade-off between safety and fuel costs, as technological advancements have made it possible to prioritize safety without incurring significant fuel cost penalties.

Therefore, option (E) weakens the argument by presenting an alternative explanation for the best-selling seat choice, which is not necessarily indicative of a higher priority on safe seating over minimizing fuel costs.

Test: Cause and Effect - Question 10

Factory manager: One reason the automobile parts this factory produces are expensive is that our manufacturing equipment is outdated and inefficient. Our products would be more competitively priced if we were to refurbish the factory completely with new, more efficient equipment. Therefore, since to survive in today’s market we have to make our products more competitively priced, we must completely refurbish the factory in order to survive.

The reasoning in the factory manager’s argument is flawed because this argument:

Detailed Solution for Test: Cause and Effect - Question 10

(A) fails to recognize that the price of a particular commodity can change over time.

This option suggests that the factory manager's argument overlooks the fact that the price of automobile parts can change over time due to various factors. It implies that the high prices may not solely be attributed to outdated equipment, but rather to other factors such as market demand, competition, or raw material costs.

(B) shifts without justification from treating something as one way of achieving a goal to treating it as the only way of achieving that goal.

This option accurately identifies the flaw in the factory manager's argument. The argument initially presents refurbishing the factory with new equipment as a way to make products more competitively priced. However, it fails to provide proper justification for why refurbishing is the only possible solution and does not consider alternative approaches to reducing costs or improving competitiveness.

(C) argues that one thing is the cause of another when the evidence given indicates that the second thing may, in fact, be the cause of the first.

This option suggests that the factory manager's argument incorrectly establishes a cause-effect relationship between outdated equipment and expensive automobile parts. It implies that there might be other factors, such as the high price of parts, leading to the inability to upgrade equipment or invest in modernization.

(D) recommends a solution to a problem without first considering any possible causes of that problem.

This option points out that the factory manager's argument immediately jumps to the solution of refurbishing the factory without properly analyzing or addressing the underlying causes of expensive automobile parts. It suggests that the argument overlooks the need to identify and evaluate other possible causes before proposing a specific solution.

(E) fails to make a definite recommendation and instead merely suggests that some possible course of action might be effective.

This option suggests that the factory manager's argument lacks a clear and decisive recommendation. It implies that the argument merely suggests that refurbishing the factory could be an effective course of action without providing strong evidence or a solid plan to support this suggestion.

Among these options, (B) best identifies the flaw in the factory manager's argument by pointing out the unjustified shift from presenting refurbishing as one way to achieve the goal to treating it as the only way to achieve that goal.

18 docs|139 tests
Information about Test: Cause and Effect Page
In this test you can find the Exam questions for Test: Cause and Effect solved & explained in the simplest way possible. Besides giving Questions and answers for Test: Cause and Effect, EduRev gives you an ample number of Online tests for practice

Top Courses for GMAT

Download as PDF

Top Courses for GMAT